Proving Hermitian Operator C is Hermitian

But if you fix the second one, the first one is unnecessary because the i has a minus sign, so its order doesn't matter. So it's really just one mistake. So you should write\begin{align}(iAB)^\dagger &= -i(BA)^\dagger = iA^\dagger B^\dagger\\(iBA)^\dagger &= -i(AB)^\dagger = -iB^\dagger A^\dagger.\end{align}In summary, to show that C=i[A,B] is hermitian, we must show that C and C^{\dagger} are equal. Using the definition of C and the calculation rule (AB)^\d
  • #1
Lindsayyyy
219
0
Hi,

Homework Statement



Let A and B be hermitian operators. Show that C=i[A,B] is hermitian aswell.



Homework Equations



-

The Attempt at a Solution



Well, I tried just to use the definition but I'm not sure if that's enough (my guess would be no lol).

[tex] C=i[A,B]=iAB-iBA=iA^\dagger B^\dagger-iB^\dagger A^\dagger =i[A^\dagger , B^\dagger ][/tex]


I'm not used to proof stuff as we don't learn it in my degree program.
Is that right, or is that a whole wrong approach?

Thanks for your help
 
Physics news on Phys.org
  • #2
[tex]C=i[A,B]=iAB-iBA = iA^{\dagger}B^{\dagger}- i B ^{\dagger} A^{\dagger}[/tex]
[tex]C^{\dagger}=(iAB-iBA)^{\dagger}[/tex]
Can you show that [itex]C[/itex] and [itex]C^{\dagger}[/itex] are the same?
 
  • #3
I'd say:

[tex] C^\dagger = (iAB)^\dagger - (iBA)^\dagger = iA^\dagger B^\dagger-iB^\dagger A^\dagger [/tex]
 
  • #4
Yup, so I guess you're done ;)
 
  • #5
Thanks for the help, but is that really it?

I can't believe that actually because it looks so trivial.

I came to the conclusion that I could show as well that the diagonal of the operator is real, but that would be much more of an afford I guess... ^^
 
  • #6
Lindsayyyy said:
I'd say:

[tex] C^\dagger = (iAB)^\dagger - (iBA)^\dagger = iA^\dagger B^\dagger-iB^\dagger A^\dagger [/tex]

You might want to give your reasoning behind that. It's easy to write down the correct answer for the wrong reason. What is [itex] (iAB)^\dagger[/itex]?
 
  • #7
Dick said:
You might want to give your reasoning behind that. It's easy to write down the correct answer for the wrong reason. What is [itex] (iAB)^\dagger[/itex]?

Hm I just used a calculation rule which says:
[tex] (AB)^\dagger = A^\dagger B^\dagger[/tex]
 
  • #8
Lindsayyyy said:
Hm I just used a calculation rule which says:
[tex] (AB)^\dagger = A^\dagger B^\dagger[/tex]
I think Dick was more interested in how you dealt with the i. It looks like you know what you're doing, but your calculation in #3 doesn't show what you were thinking. In particular, you didn't make it clear that you understand that the first term on the left of the final equality is equal to the second term on the right. So it looked like you might have made two mistakes that cancel each other out.

Edit: As you can see in posts 13-14 below, I didn't even notice that the operators are in the wrong order on the right-hand side of the quoted equality. I used the correct rule myself, but I didn't see that you (Lindsayyyy) was using the wrong one.
 
Last edited:
  • #9
Well, I don't know what to say now :blushing: . I thought it's allowed to factor out the i.
 
  • #10
Lindsayyyy said:
Well, I don't know what to say now :blushing: . I thought it's allowed to factor out the i.
If you just take it outside, both terms on the right in #3 would get the wrong sign.
 
  • #11
I guess I did it wrong then. I don't know the right answer.

edit: hold on a second. I try something

[tex] C^\dagger = (iAB)^\dagger - (iBA)^\dagger = -iA^\dagger B^\dagger +iB^\dagger A^\dagger [/tex]

would that be right?
 
  • #12
No, you had the right answer the first time. It's true that $$(iAB)^\dagger - (iBA)^\dagger = iA^\dagger B^\dagger-iB^\dagger A^\dagger,$$ but if you want to show that you know what you're doing, you have to do the two terms separately:
\begin{align}
(iAB)^\dagger &=-i(AB)^\dagger=-iB^\dagger A^\dagger\\
(-iBA)^\dagger &=i(BA)^\dagger =iA^\dagger B^\dagger.
\end{align}
 
  • #13
Lindsayyyy said:
Hm I just used a calculation rule which says:
[tex] (AB)^\dagger = A^\dagger B^\dagger[/tex]
There is no such rule.
 
  • #14
vela said:
There is no such rule.
Ahh...I didn't even see that the operators were in the wrong order on the right.
 
  • #15
vela said:
There is no such rule.

whoops, sorry. I didn't read the rule clearly. I have to drive home now, thanks for your help. I'll take a closer look tomorrow and post again if I need further help. :smile:
 
  • #16
It is now clear that you did make two mistakes that canceled each other out: You didn't change the order of the operators, and you didn't take the complex conjugate of the i.
 

FAQ: Proving Hermitian Operator C is Hermitian

1. What is a Hermitian operator?

A Hermitian operator is a type of linear operator in quantum mechanics that satisfies the Hermitian property, which states that the operator is equal to its own adjoint. In simple terms, the adjoint of a Hermitian operator is the same as the operator itself.

2. How do you prove that an operator is Hermitian?

To prove that an operator is Hermitian, you need to show that it is equal to its own adjoint. This can be done by taking the adjoint of the operator and showing that it is the same as the original operator. This can be done using mathematical techniques, such as using the properties of matrices or using the definition of the adjoint.

3. Why is it important to prove that an operator is Hermitian?

Proving that an operator is Hermitian is important in quantum mechanics because Hermitian operators have special properties that make them useful in calculations and predictions. For example, Hermitian operators have real eigenvalues, which correspond to measurable quantities in quantum systems.

4. Can any operator be proven to be Hermitian?

No, not all operators can be proven to be Hermitian. Only operators that satisfy the Hermitian property, which states that the operator is equal to its own adjoint, can be considered Hermitian. Some operators, such as non-linear operators, do not satisfy this property and therefore cannot be proven to be Hermitian.

5. How does proving Hermitian operator C is Hermitian impact quantum mechanics?

Proving that an operator is Hermitian has a major impact on quantum mechanics because it allows us to make predictions and calculations about quantum systems. By showing that an operator is Hermitian, we can be confident that it will have real eigenvalues and can be used to make accurate measurements and predictions in quantum systems.

Back
Top